LSAT Community
Help -
Could you please explain the reason why (E) shouldn't be the answer?
Thanks,
Batman July 10, 2014
- Exam Questions
- Reading Comp
- Humanities
- June 1997 - Sec 1 - Q8
0 likes • 2 comments
Help -
How could both approach be complement each other according to the passage? I don't understand tha...
Batman July 10, 2014
- Exam Questions
- Reading Comp
- Humanities
- June 1997 - Sec 1 - Q2
0 likes • 2 comments
Why is this B and not D? -
Will you please break this problem down for me.
OffshoreSuge July 10, 2014
- Exam Questions
- Logical Reasoning
- Must Be True
- October 2011 - Sec 3 - Q18
0 likes • 1 comment
Don't quite understand -
Could you explain why the correct answer is B? Thank you!
TTabriz July 9, 2014
- Exam Questions
- Logical Reasoning
- Argument Structure
0 likes • 1 comment
Why? -
I got the right answer but that was because it seemed like the best choice. The question asks, t...
Derek July 8, 2014
- Exam Questions
- Logic Games
- Group Games
0 likes • 1 comment
I am still not understanding this question -
The question stem asks what Must be True, so what has to be true given the information. Answer B ...
Derek July 8, 2014
- Exam Questions
- Logic Games
- Group Games
0 likes • 3 comments
Explain -
Is it because since it is not legally impermissible it is legally permissible, making a legally p...
Jborj653 July 8, 2014
- Exam Questions
- Logical Reasoning
- Cannot Be True
0 likes • 7 comments
Explain -
Yeah I was confused on this one as well. I ended up going with e but I knew the connection betwee...
Jborj653 July 8, 2014
- Exam Questions
- Logical Reasoning
- Cannot Be True
0 likes • 1 comment
Why A? -
First of all it says "fan members" not "family members" but I think after you read it a couple ti...
Derek July 8, 2014
- Exam Questions
- Logical Reasoning
- Cannot Be True
0 likes • 9 comments
Assistance please -
Hey guys,
Is it possible to see a set-up for this game?
Titan July 7, 2014
- Exam Questions
- Logic Games
- The Lost Boys
0 likes • 1 comment